Real Analysis - Continuity











up vote
3
down vote

favorite












a. Give an example of a function defined everywhere on the interval $[0,1]$, which does not achieve its maximum.



b. Give an example of a function defined on $mathbb{R}$, that is nowhere continuous.



c. Give an example of a continuous function defined on a bounded set, which is not uniformly continuous.



My answers are




  1. $f(x) = x^2$


  2. $f(x) = {1$, if $x$ is rational; $-1$, if $x$ is irrational$}$

  3. $f(x) = 1/x$


My workings are attached. Please help verify if the working is correct.Solutions










share|cite|improve this question


















  • 4




    2. is correct. For 3, you need to specify the bounded domain. Your answer to 1. is incorrect, as the function certainly achieves its maximum on the interval $[0,1]$.
    – Ted Shifrin
    Dec 1 at 22:41















up vote
3
down vote

favorite












a. Give an example of a function defined everywhere on the interval $[0,1]$, which does not achieve its maximum.



b. Give an example of a function defined on $mathbb{R}$, that is nowhere continuous.



c. Give an example of a continuous function defined on a bounded set, which is not uniformly continuous.



My answers are




  1. $f(x) = x^2$


  2. $f(x) = {1$, if $x$ is rational; $-1$, if $x$ is irrational$}$

  3. $f(x) = 1/x$


My workings are attached. Please help verify if the working is correct.Solutions










share|cite|improve this question


















  • 4




    2. is correct. For 3, you need to specify the bounded domain. Your answer to 1. is incorrect, as the function certainly achieves its maximum on the interval $[0,1]$.
    – Ted Shifrin
    Dec 1 at 22:41













up vote
3
down vote

favorite









up vote
3
down vote

favorite











a. Give an example of a function defined everywhere on the interval $[0,1]$, which does not achieve its maximum.



b. Give an example of a function defined on $mathbb{R}$, that is nowhere continuous.



c. Give an example of a continuous function defined on a bounded set, which is not uniformly continuous.



My answers are




  1. $f(x) = x^2$


  2. $f(x) = {1$, if $x$ is rational; $-1$, if $x$ is irrational$}$

  3. $f(x) = 1/x$


My workings are attached. Please help verify if the working is correct.Solutions










share|cite|improve this question













a. Give an example of a function defined everywhere on the interval $[0,1]$, which does not achieve its maximum.



b. Give an example of a function defined on $mathbb{R}$, that is nowhere continuous.



c. Give an example of a continuous function defined on a bounded set, which is not uniformly continuous.



My answers are




  1. $f(x) = x^2$


  2. $f(x) = {1$, if $x$ is rational; $-1$, if $x$ is irrational$}$

  3. $f(x) = 1/x$


My workings are attached. Please help verify if the working is correct.Solutions







real-analysis continuity maxima-minima uniform-continuity






share|cite|improve this question













share|cite|improve this question











share|cite|improve this question




share|cite|improve this question










asked Dec 1 at 22:36









Ty Johnson

283




283








  • 4




    2. is correct. For 3, you need to specify the bounded domain. Your answer to 1. is incorrect, as the function certainly achieves its maximum on the interval $[0,1]$.
    – Ted Shifrin
    Dec 1 at 22:41














  • 4




    2. is correct. For 3, you need to specify the bounded domain. Your answer to 1. is incorrect, as the function certainly achieves its maximum on the interval $[0,1]$.
    – Ted Shifrin
    Dec 1 at 22:41








4




4




2. is correct. For 3, you need to specify the bounded domain. Your answer to 1. is incorrect, as the function certainly achieves its maximum on the interval $[0,1]$.
– Ted Shifrin
Dec 1 at 22:41




2. is correct. For 3, you need to specify the bounded domain. Your answer to 1. is incorrect, as the function certainly achieves its maximum on the interval $[0,1]$.
– Ted Shifrin
Dec 1 at 22:41










3 Answers
3






active

oldest

votes

















up vote
6
down vote



accepted










$a)$ Consider $f(x)$ on $[0,1]$ such that $f(0) = 0, f(1) = 0, f(x) = dfrac{1}{x}, 0 <x < 1$.



$b)$ The function you had is a good one.



$c)$ Consider $f(x)$ on $(0,1)$ and $f(x) = dfrac{1}{x}$.






share|cite|improve this answer




























    up vote
    4
    down vote














    1. Wrong: the maximum is $1$, which is $f(1)$.

    2. Right.

    3. Your answer is incomplete, at best, since you did not state what is the domain of your function.






    share|cite|improve this answer




























      up vote
      2
      down vote













      1)



      Extreme value theorem says that every continuous function over a closed interval has a maximum.



      You need a function that is not continuous.



      $f(x) = begin{cases} f(x) = x & x<1\f(x) = 0 & x=1 end{cases}$






      share|cite|improve this answer





















        Your Answer





        StackExchange.ifUsing("editor", function () {
        return StackExchange.using("mathjaxEditing", function () {
        StackExchange.MarkdownEditor.creationCallbacks.add(function (editor, postfix) {
        StackExchange.mathjaxEditing.prepareWmdForMathJax(editor, postfix, [["$", "$"], ["\\(","\\)"]]);
        });
        });
        }, "mathjax-editing");

        StackExchange.ready(function() {
        var channelOptions = {
        tags: "".split(" "),
        id: "69"
        };
        initTagRenderer("".split(" "), "".split(" "), channelOptions);

        StackExchange.using("externalEditor", function() {
        // Have to fire editor after snippets, if snippets enabled
        if (StackExchange.settings.snippets.snippetsEnabled) {
        StackExchange.using("snippets", function() {
        createEditor();
        });
        }
        else {
        createEditor();
        }
        });

        function createEditor() {
        StackExchange.prepareEditor({
        heartbeatType: 'answer',
        convertImagesToLinks: true,
        noModals: true,
        showLowRepImageUploadWarning: true,
        reputationToPostImages: 10,
        bindNavPrevention: true,
        postfix: "",
        imageUploader: {
        brandingHtml: "Powered by u003ca class="icon-imgur-white" href="https://imgur.com/"u003eu003c/au003e",
        contentPolicyHtml: "User contributions licensed under u003ca href="https://creativecommons.org/licenses/by-sa/3.0/"u003ecc by-sa 3.0 with attribution requiredu003c/au003e u003ca href="https://stackoverflow.com/legal/content-policy"u003e(content policy)u003c/au003e",
        allowUrls: true
        },
        noCode: true, onDemand: true,
        discardSelector: ".discard-answer"
        ,immediatelyShowMarkdownHelp:true
        });


        }
        });














        draft saved

        draft discarded


















        StackExchange.ready(
        function () {
        StackExchange.openid.initPostLogin('.new-post-login', 'https%3a%2f%2fmath.stackexchange.com%2fquestions%2f3021931%2freal-analysis-continuity%23new-answer', 'question_page');
        }
        );

        Post as a guest















        Required, but never shown

























        3 Answers
        3






        active

        oldest

        votes








        3 Answers
        3






        active

        oldest

        votes









        active

        oldest

        votes






        active

        oldest

        votes








        up vote
        6
        down vote



        accepted










        $a)$ Consider $f(x)$ on $[0,1]$ such that $f(0) = 0, f(1) = 0, f(x) = dfrac{1}{x}, 0 <x < 1$.



        $b)$ The function you had is a good one.



        $c)$ Consider $f(x)$ on $(0,1)$ and $f(x) = dfrac{1}{x}$.






        share|cite|improve this answer

























          up vote
          6
          down vote



          accepted










          $a)$ Consider $f(x)$ on $[0,1]$ such that $f(0) = 0, f(1) = 0, f(x) = dfrac{1}{x}, 0 <x < 1$.



          $b)$ The function you had is a good one.



          $c)$ Consider $f(x)$ on $(0,1)$ and $f(x) = dfrac{1}{x}$.






          share|cite|improve this answer























            up vote
            6
            down vote



            accepted







            up vote
            6
            down vote



            accepted






            $a)$ Consider $f(x)$ on $[0,1]$ such that $f(0) = 0, f(1) = 0, f(x) = dfrac{1}{x}, 0 <x < 1$.



            $b)$ The function you had is a good one.



            $c)$ Consider $f(x)$ on $(0,1)$ and $f(x) = dfrac{1}{x}$.






            share|cite|improve this answer












            $a)$ Consider $f(x)$ on $[0,1]$ such that $f(0) = 0, f(1) = 0, f(x) = dfrac{1}{x}, 0 <x < 1$.



            $b)$ The function you had is a good one.



            $c)$ Consider $f(x)$ on $(0,1)$ and $f(x) = dfrac{1}{x}$.







            share|cite|improve this answer












            share|cite|improve this answer



            share|cite|improve this answer










            answered Dec 1 at 22:50









            DeepSea

            70.8k54487




            70.8k54487






















                up vote
                4
                down vote














                1. Wrong: the maximum is $1$, which is $f(1)$.

                2. Right.

                3. Your answer is incomplete, at best, since you did not state what is the domain of your function.






                share|cite|improve this answer

























                  up vote
                  4
                  down vote














                  1. Wrong: the maximum is $1$, which is $f(1)$.

                  2. Right.

                  3. Your answer is incomplete, at best, since you did not state what is the domain of your function.






                  share|cite|improve this answer























                    up vote
                    4
                    down vote










                    up vote
                    4
                    down vote










                    1. Wrong: the maximum is $1$, which is $f(1)$.

                    2. Right.

                    3. Your answer is incomplete, at best, since you did not state what is the domain of your function.






                    share|cite|improve this answer













                    1. Wrong: the maximum is $1$, which is $f(1)$.

                    2. Right.

                    3. Your answer is incomplete, at best, since you did not state what is the domain of your function.







                    share|cite|improve this answer












                    share|cite|improve this answer



                    share|cite|improve this answer










                    answered Dec 1 at 22:44









                    José Carlos Santos

                    147k22117217




                    147k22117217






















                        up vote
                        2
                        down vote













                        1)



                        Extreme value theorem says that every continuous function over a closed interval has a maximum.



                        You need a function that is not continuous.



                        $f(x) = begin{cases} f(x) = x & x<1\f(x) = 0 & x=1 end{cases}$






                        share|cite|improve this answer

























                          up vote
                          2
                          down vote













                          1)



                          Extreme value theorem says that every continuous function over a closed interval has a maximum.



                          You need a function that is not continuous.



                          $f(x) = begin{cases} f(x) = x & x<1\f(x) = 0 & x=1 end{cases}$






                          share|cite|improve this answer























                            up vote
                            2
                            down vote










                            up vote
                            2
                            down vote









                            1)



                            Extreme value theorem says that every continuous function over a closed interval has a maximum.



                            You need a function that is not continuous.



                            $f(x) = begin{cases} f(x) = x & x<1\f(x) = 0 & x=1 end{cases}$






                            share|cite|improve this answer












                            1)



                            Extreme value theorem says that every continuous function over a closed interval has a maximum.



                            You need a function that is not continuous.



                            $f(x) = begin{cases} f(x) = x & x<1\f(x) = 0 & x=1 end{cases}$







                            share|cite|improve this answer












                            share|cite|improve this answer



                            share|cite|improve this answer










                            answered Dec 1 at 23:27









                            Doug M

                            43.5k31854




                            43.5k31854






























                                draft saved

                                draft discarded




















































                                Thanks for contributing an answer to Mathematics Stack Exchange!


                                • Please be sure to answer the question. Provide details and share your research!

                                But avoid



                                • Asking for help, clarification, or responding to other answers.

                                • Making statements based on opinion; back them up with references or personal experience.


                                Use MathJax to format equations. MathJax reference.


                                To learn more, see our tips on writing great answers.





                                Some of your past answers have not been well-received, and you're in danger of being blocked from answering.


                                Please pay close attention to the following guidance:


                                • Please be sure to answer the question. Provide details and share your research!

                                But avoid



                                • Asking for help, clarification, or responding to other answers.

                                • Making statements based on opinion; back them up with references or personal experience.


                                To learn more, see our tips on writing great answers.




                                draft saved


                                draft discarded














                                StackExchange.ready(
                                function () {
                                StackExchange.openid.initPostLogin('.new-post-login', 'https%3a%2f%2fmath.stackexchange.com%2fquestions%2f3021931%2freal-analysis-continuity%23new-answer', 'question_page');
                                }
                                );

                                Post as a guest















                                Required, but never shown





















































                                Required, but never shown














                                Required, but never shown












                                Required, but never shown







                                Required, but never shown

































                                Required, but never shown














                                Required, but never shown












                                Required, but never shown







                                Required, but never shown







                                Popular posts from this blog

                                Quarter-circle Tiles

                                build a pushdown automaton that recognizes the reverse language of a given pushdown automaton?

                                Mont Emei